Skip to main content

Stack Exchange Network

Stack Exchange network consists of 183 Q&A communities including Stack Overflow, the largest, most trusted online community for developers to learn, share their knowledge, and build their careers.

Visit Stack Exchange

You are not logged in. Your edit will be placed in a queue until it is peer reviewed.

We welcome edits that make the post easier to understand and more valuable for readers. Because community members review edits, please try to make the post substantially better than how you found it, for example, by fixing grammar or adding additional resources and hyperlinks.

Required fields*

Required fields*

A curious Slitherlink Deduction

In the following Slitherlink, where you need to make one unbroken loop over the grid with the number in each cell matching the number of edges around that cell, I noticed that the 3-2-2-1 2x2 box at the middle top has only one possibility: the 1 having their right edge on as opposed to their bottom edge.

partially filled Slitherlink board screenshot

It's a quick deduction if you do trial and error using the bottom edge of the 1, as it quickly leads to contradiction on the top two cells.

But as trial and error method is typically not the first method one would use, I'm curious whether there are another way to deduce that the 1 should have the right edge on, instead of the bottom edge. My usual method of making an inaginary loop and counting edges (must be even) or even the latest trick I learned from this community about inside-outside don't seem to work 🤔

(Or otherwise, how to proceed in from this partially filled grid. I tried trial and error on the bottom left too, but ended up needing to resolve this section as well.)

Answer*

Cancel
4
  • $\begingroup$ Ah thanks! I missed that loop through the 2. $\endgroup$
    justhalf
    –  justhalf
    2025-10-13 10:51:06 +00:00
    Commented yesterday
  • $\begingroup$ This certainly works but makes use of a lot more of the board. With trial and error one can already deduce the contradiction in the 2x2 box by only considering the box and the 4 edges going into it. But I don't know whether there is any deduction to apply here that is more general than this specific configuration. $\endgroup$
    quarague
    –  quarague
    2025-10-14 06:32:30 +00:00
    Commented 23 hours ago
  • $\begingroup$ @quarague The only additional area used in this case is the immediately adjacent ring of cella. (I may have coloured a few more in that image, but they don't actually assist the solution.) Those adjacent cells have to be included in any deduction you make here, because the lines entering/leaving the 2x2 affect which internal configurations are valid. $\endgroup$
    fljx
    –  fljx
    2025-10-14 06:45:23 +00:00
    Commented 23 hours ago
  • $\begingroup$ If you only keep all the lines in the 4x3 block surrounding the 2x2 block and leave the rest of the grid empty, then I think trial-and-error still allows to deduce the edge on the 1 but your deduction wouldn't work anymore. $\endgroup$
    quarague
    –  quarague
    2025-10-14 06:48:45 +00:00
    Commented 23 hours ago

Morty Proxy This is a proxified and sanitized view of the page, visit original site.